1
$\begingroup$

After reading "Mathematics for physicists" by Susan M. Lea I encountered a subtlety that I can't turn my head around (p. 128). Consider function

$$f(z)=\frac{1}{z^2-1} = \frac{1}{2}\left[\frac{1}{z-1}-\frac{1}{1+z}\right]$$

which clearly has 2 simple poles at $z=\pm 1$. If we want to compute its Laurent series around $z=1$, we would get:

$$\frac{1}{1+z}=\frac{1}{2+(z-1)} = \frac{1}{(z-1)\left(1+\frac{2}{z-1}\right)}$$ $$=\frac{1}{z-1}\sum_{n=0}^\infty\frac{(-2)^n}{(z-1)^{n}}$$ and thus: $$\frac{1}{z^2-1} = \frac{1}{2}\left[\frac{1}{z-1}-\sum_{n=0}(-1)^n\frac{2^n}{(z-1)^{n+1}}\right] = \frac{1}{2}\sum_{n=1}^\infty\frac{(-2)^n}{(z-1)^{n+1}}$$

What I see here is an infinite number of negative powers of (z-1) and thus, I would be tempted to conclude that there is an essential singularity at this point. What prevents me from concluding this as it is clear that there is no essential singularity from the analytic function?

$\endgroup$
3
  • 1
    $\begingroup$ What is your radius of convergence? $\endgroup$
    – user765629
    Commented Oct 29, 2020 at 2:59
  • $\begingroup$ I would say that the radius of convergence is 2 in this case since there is a pole at $z=-1$. $\endgroup$
    – fgoudra
    Commented Oct 29, 2020 at 3:03
  • 2
    $\begingroup$ It only converges if $|z-1|>2$. i.e. the sum does not exist near your point of interest. $\endgroup$
    – user765629
    Commented Oct 29, 2020 at 3:14

1 Answer 1

2
$\begingroup$

As I mentioned in the comments, the series you derived does not exists in a punctured neighborhood of the point $z=1$ as it only converges in $|z-1|>2$.

You should have expanded $\frac{1}{1+z}$ via $$\frac{1}{1+z}=\frac{1}{2-(1-z)}=\frac{1}{2}\frac{1}{1-\frac{1-z}{2}}=\frac12\sum_{n=0}^\infty\left( \frac{1-z}{2}\right)^n$$ and it converges in $|z-1|<2$. So about $z=1$ your full Laurent series is $$f(z)=\frac{1}{2}\left[-\frac{1}{1-z}-\frac12\sum_{n=0}^\infty \left(\frac{1-z}{2}\right)^n \right]=-\frac{1}{4}\sum_{n=-1}^\infty \left(\frac{1-z}{2}\right)^n. $$

$\endgroup$
1
  • $\begingroup$ yes that is correct. you should add what was stated by @epiliam, the Laurent series I stated in the question is not valid near z=1. If you could add this to your answer I will mark it as the answer. $\endgroup$
    – fgoudra
    Commented Oct 29, 2020 at 14:06

You must log in to answer this question.

Not the answer you're looking for? Browse other questions tagged .